11.01.2014 Views

1 Calculus III Exam 2 Practice Problems - Solutions

1 Calculus III Exam 2 Practice Problems - Solutions

1 Calculus III Exam 2 Practice Problems - Solutions

SHOW MORE
SHOW LESS

Create successful ePaper yourself

Turn your PDF publications into a flip-book with our unique Google optimized e-Paper software.

<strong>Calculus</strong> <strong>III</strong>, <strong>Exam</strong> 2: Review <strong>Solutions</strong> 2<br />

Computing d x = 4x + 2y − 12 and d y = 2x + 4y − 8, we see both exist for all values of x, y, so<br />

the only critical points of d(x, y) occur where d x = d y = 0. Moreover, as d is defined for all (x, y),<br />

there is no boundary to consider, so any extreme points occur at critical points. Now d x = 0<br />

implies y = 6 − 2x. Plugging this in to the equation d y = 0 we obtain 2x + 4(6 − 2x) − 8 = 16 − 6x,<br />

so x = 8/3, y = 2/3, z = 1/3. Hence (8/3, 2/3, 1/3) is the point on the plane closest to (1, −1, 2).<br />

(The second derivative test, with d xx = 4, d xy = 2, d yy = 4, implies that D = 4 2 − 2 2 > 0 and<br />

d xx > 0 proves this is a minimum).<br />

6. Evaluate the following expressions.<br />

(a)<br />

lim<br />

(x,y)→(0,0)<br />

2 + y<br />

x + cos y = lim 2+y<br />

(0,0) x+cos y = 2+0<br />

0+1 = 2<br />

∂<br />

(b)<br />

∂x [tan−1 (x 2 y) + tan −1 (xy 2 )] =<br />

2xy<br />

y<br />

1+(x 2 y)<br />

+<br />

2<br />

2 1+(xy 2 ) 2<br />

(c) f xy (0, 1) where f(x, y) = (x 3 + x)y 2 : f x = (3x 2 + 1)y 2 so f xy = (3x 2 + 1)(2y) and f xy (0, 1) =<br />

(1)(2) = 2<br />

(d) ∇(x 3 y 4 ) = 〈3x 2 y 4 , 4x 3 y 3 〉<br />

(e) f x (2, 4) where f(x, y) = xe y/x : f x = e y/x + xe y/x (−yx −2 ) so f x (2, 4) = e 2 + 2e 2 (−1)<br />

(f) D ⃗u (x 2 − y 3 ) where ⃗u = 1 2 ⃗ı − √ 3<br />

2 ⃗j: D ⃗u(x 2 − y 3 ) = 〈2x, −3y 2 〉 · 〈1/2, √ 3/2〉 = x − (3 √ 3/2)y 2<br />

7. Find the local max, local min, and saddle points of f(x, y) = 4xy − x 4 − y 4 . Does f have any<br />

absolute extreme values?<br />

Solution: f x = 4y −4x 3 = 0 when y = x 3 . f y = 4x−4y 3 = 0 when x = y 3 . Combining these two,<br />

we need y = (y 3 ) 3 = y 9 , which only happens if y = 0, 1, −1 and so x = 0, 1, −1 correspondingly.<br />

That is, the only critical points of f are (0, 0), (1, 1), (−1, −1) (since f x , f y are defined for all x, y).<br />

Now apply SDT: f xx = −12x 2 , f xy = 4, f yy = −12y 2 , so<br />

∆(0, 0) = (0)(0) − (4) 2 < 0 =⇒ saddle point<br />

∆(1, 1) = (−12)(−12) − (4) 2 > 0 and f xx (1, 1) = −12 =⇒ local max<br />

∆(−1, −1) = (−12)(−12) − (4) 2 > 0 and f xx (−1, −1) = −12 =⇒ local max<br />

Lastly, f has an absolute max at both (1, 1) and (−1, −1), but it doesn’t have an absolute min,<br />

for as x 2 + y 2 → ∞, f → −∞.<br />

8. Find the directions of maximum increase and decrease for f(x, y) = x 2/3 + xy + y 2/3 at (−1, 1).<br />

What are the corresponding magnitudes of these changes?<br />

Solution: Max increase occurs in the direction of ∇f(−1, 1) = 〈2/(3x 1/3 )+y, x+2/(3y 1/3 )〉(−1, 1) =<br />

〈−2/3+1, −1+2/3〉 = 〈1/3, −1/3〉 and the magnitude of change in this direction is ||∇f(−1, 1)|| =<br />

√<br />

2/9. The greatest decrease occurs in the direction −∇f(−1, 1) = 〈−1/3, 1/3〉 with corresponding<br />

change − √ 2/9.<br />

9. Find ∂w<br />

∂s<br />

and<br />

∂w<br />

∂t given w = x2 + xy + y 2 with x = 2r + s and y = r − 2s.<br />

∂w<br />

Solution:<br />

∂r<br />

= ∂w<br />

∂x<br />

y)(1) + (x + 2y)(−2).<br />

∂x<br />

∂r + ∂w<br />

∂y<br />

∂y<br />

∂r<br />

∂w<br />

= (2x + y)(2) + (x + 2y)(1) and<br />

∂s<br />

= ∂w ∂x<br />

∂x ∂s + ∂w ∂y<br />

∂y ∂s = (2x +<br />

10. Find the extreme values of f(x, y) = xy in the region bounded by the positive x-axis, the positive<br />

y-axis, and the circle x 2 + y 2 − 10 = 0.<br />

Solution: First we look for critical points of f: f x = y = 0 and f y = x = 0 only at the origin,<br />

(0, 0), which is not an interior point of the domain, so the book and most people (including myself)<br />

will disagree in calling this a critical point. Technically, it is a boundary point of the domain, and<br />

so is not a critical point, but if you call it one, I won’t hurt you. What this does say, however,<br />

is that the function can only assume its max and min values on the boundary. Let’s consider

Hooray! Your file is uploaded and ready to be published.

Saved successfully!

Ooh no, something went wrong!